show that the limit does not exist: $lim_{xto0}frac{1}{x^2}$












1












$begingroup$



Show that the following limit does not exist:
$$lim_{xto 0}frac{1}{x^2}$$
$(x>0)$





The $delta$ - $varepsilon$ definition can be used to prove a given limit exists for some function at a particular point. My question is, can we prove the non-existence of a limit using the $delta$ - $varepsilon$ definition? (A little hint on how, if yes.)

Besides that, what (other) methods can we use?




[SIMILAR POST: Can we prove that there is no limit at $x=0$ for $f(x)=1/x$ using epsilon-delta definition? ]










share|cite|improve this question











$endgroup$








  • 2




    $begingroup$
    It does exist though.
    $endgroup$
    – Rebellos
    Dec 20 '18 at 9:59






  • 1




    $begingroup$
    The limit exists but is $+infty$: to see this consider simply $limsup$ and $liminf$: the same is true even if you consider the whole $mathbb{R}$ instead of the positive axis. Perhaps the standard terminology is a bit hazy since in such cases it is said that the function diverges: however this does not mean tha the limit does not exists, but simply that the limit is $infty$.
    $endgroup$
    – Daniele Tampieri
    Dec 20 '18 at 10:00






  • 1




    $begingroup$
    Show that for every $L>0$ and $varepsilon=1;exists:delta>0$, $|f(x)-L|>varepsilon$ whenever $|x|<delta$.
    $endgroup$
    – Yadati Kiran
    Dec 20 '18 at 10:00


















1












$begingroup$



Show that the following limit does not exist:
$$lim_{xto 0}frac{1}{x^2}$$
$(x>0)$





The $delta$ - $varepsilon$ definition can be used to prove a given limit exists for some function at a particular point. My question is, can we prove the non-existence of a limit using the $delta$ - $varepsilon$ definition? (A little hint on how, if yes.)

Besides that, what (other) methods can we use?




[SIMILAR POST: Can we prove that there is no limit at $x=0$ for $f(x)=1/x$ using epsilon-delta definition? ]










share|cite|improve this question











$endgroup$








  • 2




    $begingroup$
    It does exist though.
    $endgroup$
    – Rebellos
    Dec 20 '18 at 9:59






  • 1




    $begingroup$
    The limit exists but is $+infty$: to see this consider simply $limsup$ and $liminf$: the same is true even if you consider the whole $mathbb{R}$ instead of the positive axis. Perhaps the standard terminology is a bit hazy since in such cases it is said that the function diverges: however this does not mean tha the limit does not exists, but simply that the limit is $infty$.
    $endgroup$
    – Daniele Tampieri
    Dec 20 '18 at 10:00






  • 1




    $begingroup$
    Show that for every $L>0$ and $varepsilon=1;exists:delta>0$, $|f(x)-L|>varepsilon$ whenever $|x|<delta$.
    $endgroup$
    – Yadati Kiran
    Dec 20 '18 at 10:00
















1












1








1





$begingroup$



Show that the following limit does not exist:
$$lim_{xto 0}frac{1}{x^2}$$
$(x>0)$





The $delta$ - $varepsilon$ definition can be used to prove a given limit exists for some function at a particular point. My question is, can we prove the non-existence of a limit using the $delta$ - $varepsilon$ definition? (A little hint on how, if yes.)

Besides that, what (other) methods can we use?




[SIMILAR POST: Can we prove that there is no limit at $x=0$ for $f(x)=1/x$ using epsilon-delta definition? ]










share|cite|improve this question











$endgroup$





Show that the following limit does not exist:
$$lim_{xto 0}frac{1}{x^2}$$
$(x>0)$





The $delta$ - $varepsilon$ definition can be used to prove a given limit exists for some function at a particular point. My question is, can we prove the non-existence of a limit using the $delta$ - $varepsilon$ definition? (A little hint on how, if yes.)

Besides that, what (other) methods can we use?




[SIMILAR POST: Can we prove that there is no limit at $x=0$ for $f(x)=1/x$ using epsilon-delta definition? ]







real-analysis limits






share|cite|improve this question















share|cite|improve this question













share|cite|improve this question




share|cite|improve this question








edited Dec 20 '18 at 10:14







Za Ira

















asked Dec 20 '18 at 9:51









Za IraZa Ira

161115




161115








  • 2




    $begingroup$
    It does exist though.
    $endgroup$
    – Rebellos
    Dec 20 '18 at 9:59






  • 1




    $begingroup$
    The limit exists but is $+infty$: to see this consider simply $limsup$ and $liminf$: the same is true even if you consider the whole $mathbb{R}$ instead of the positive axis. Perhaps the standard terminology is a bit hazy since in such cases it is said that the function diverges: however this does not mean tha the limit does not exists, but simply that the limit is $infty$.
    $endgroup$
    – Daniele Tampieri
    Dec 20 '18 at 10:00






  • 1




    $begingroup$
    Show that for every $L>0$ and $varepsilon=1;exists:delta>0$, $|f(x)-L|>varepsilon$ whenever $|x|<delta$.
    $endgroup$
    – Yadati Kiran
    Dec 20 '18 at 10:00
















  • 2




    $begingroup$
    It does exist though.
    $endgroup$
    – Rebellos
    Dec 20 '18 at 9:59






  • 1




    $begingroup$
    The limit exists but is $+infty$: to see this consider simply $limsup$ and $liminf$: the same is true even if you consider the whole $mathbb{R}$ instead of the positive axis. Perhaps the standard terminology is a bit hazy since in such cases it is said that the function diverges: however this does not mean tha the limit does not exists, but simply that the limit is $infty$.
    $endgroup$
    – Daniele Tampieri
    Dec 20 '18 at 10:00






  • 1




    $begingroup$
    Show that for every $L>0$ and $varepsilon=1;exists:delta>0$, $|f(x)-L|>varepsilon$ whenever $|x|<delta$.
    $endgroup$
    – Yadati Kiran
    Dec 20 '18 at 10:00










2




2




$begingroup$
It does exist though.
$endgroup$
– Rebellos
Dec 20 '18 at 9:59




$begingroup$
It does exist though.
$endgroup$
– Rebellos
Dec 20 '18 at 9:59




1




1




$begingroup$
The limit exists but is $+infty$: to see this consider simply $limsup$ and $liminf$: the same is true even if you consider the whole $mathbb{R}$ instead of the positive axis. Perhaps the standard terminology is a bit hazy since in such cases it is said that the function diverges: however this does not mean tha the limit does not exists, but simply that the limit is $infty$.
$endgroup$
– Daniele Tampieri
Dec 20 '18 at 10:00




$begingroup$
The limit exists but is $+infty$: to see this consider simply $limsup$ and $liminf$: the same is true even if you consider the whole $mathbb{R}$ instead of the positive axis. Perhaps the standard terminology is a bit hazy since in such cases it is said that the function diverges: however this does not mean tha the limit does not exists, but simply that the limit is $infty$.
$endgroup$
– Daniele Tampieri
Dec 20 '18 at 10:00




1




1




$begingroup$
Show that for every $L>0$ and $varepsilon=1;exists:delta>0$, $|f(x)-L|>varepsilon$ whenever $|x|<delta$.
$endgroup$
– Yadati Kiran
Dec 20 '18 at 10:00






$begingroup$
Show that for every $L>0$ and $varepsilon=1;exists:delta>0$, $|f(x)-L|>varepsilon$ whenever $|x|<delta$.
$endgroup$
– Yadati Kiran
Dec 20 '18 at 10:00












1 Answer
1






active

oldest

votes


















6












$begingroup$

What is asserted is that there is no finite limit. Prove by contradiction. Suppose $lim_{xto 0+}frac 1 {x^{2}}=L$ exists and is finite. Then we can find $delta >0$ such that $|frac 1 {x^{2}}-L|<1$ whenever $0<x<delta$. If $n$ is any sufficiently large integer then $0<frac 1 n<delta$ so, taking $x=frac 1 n$ we get $| n^{2}-L|<1$. This gives $n^{2} <L+1$ for any sufficiently large integer $n$ which is obviously a contradiction.






share|cite|improve this answer









$endgroup$













    Your Answer





    StackExchange.ifUsing("editor", function () {
    return StackExchange.using("mathjaxEditing", function () {
    StackExchange.MarkdownEditor.creationCallbacks.add(function (editor, postfix) {
    StackExchange.mathjaxEditing.prepareWmdForMathJax(editor, postfix, [["$", "$"], ["\\(","\\)"]]);
    });
    });
    }, "mathjax-editing");

    StackExchange.ready(function() {
    var channelOptions = {
    tags: "".split(" "),
    id: "69"
    };
    initTagRenderer("".split(" "), "".split(" "), channelOptions);

    StackExchange.using("externalEditor", function() {
    // Have to fire editor after snippets, if snippets enabled
    if (StackExchange.settings.snippets.snippetsEnabled) {
    StackExchange.using("snippets", function() {
    createEditor();
    });
    }
    else {
    createEditor();
    }
    });

    function createEditor() {
    StackExchange.prepareEditor({
    heartbeatType: 'answer',
    autoActivateHeartbeat: false,
    convertImagesToLinks: true,
    noModals: true,
    showLowRepImageUploadWarning: true,
    reputationToPostImages: 10,
    bindNavPrevention: true,
    postfix: "",
    imageUploader: {
    brandingHtml: "Powered by u003ca class="icon-imgur-white" href="https://imgur.com/"u003eu003c/au003e",
    contentPolicyHtml: "User contributions licensed under u003ca href="https://creativecommons.org/licenses/by-sa/3.0/"u003ecc by-sa 3.0 with attribution requiredu003c/au003e u003ca href="https://stackoverflow.com/legal/content-policy"u003e(content policy)u003c/au003e",
    allowUrls: true
    },
    noCode: true, onDemand: true,
    discardSelector: ".discard-answer"
    ,immediatelyShowMarkdownHelp:true
    });


    }
    });














    draft saved

    draft discarded


















    StackExchange.ready(
    function () {
    StackExchange.openid.initPostLogin('.new-post-login', 'https%3a%2f%2fmath.stackexchange.com%2fquestions%2f3047354%2fshow-that-the-limit-does-not-exist-lim-x-to0-frac1x2%23new-answer', 'question_page');
    }
    );

    Post as a guest















    Required, but never shown

























    1 Answer
    1






    active

    oldest

    votes








    1 Answer
    1






    active

    oldest

    votes









    active

    oldest

    votes






    active

    oldest

    votes









    6












    $begingroup$

    What is asserted is that there is no finite limit. Prove by contradiction. Suppose $lim_{xto 0+}frac 1 {x^{2}}=L$ exists and is finite. Then we can find $delta >0$ such that $|frac 1 {x^{2}}-L|<1$ whenever $0<x<delta$. If $n$ is any sufficiently large integer then $0<frac 1 n<delta$ so, taking $x=frac 1 n$ we get $| n^{2}-L|<1$. This gives $n^{2} <L+1$ for any sufficiently large integer $n$ which is obviously a contradiction.






    share|cite|improve this answer









    $endgroup$


















      6












      $begingroup$

      What is asserted is that there is no finite limit. Prove by contradiction. Suppose $lim_{xto 0+}frac 1 {x^{2}}=L$ exists and is finite. Then we can find $delta >0$ such that $|frac 1 {x^{2}}-L|<1$ whenever $0<x<delta$. If $n$ is any sufficiently large integer then $0<frac 1 n<delta$ so, taking $x=frac 1 n$ we get $| n^{2}-L|<1$. This gives $n^{2} <L+1$ for any sufficiently large integer $n$ which is obviously a contradiction.






      share|cite|improve this answer









      $endgroup$
















        6












        6








        6





        $begingroup$

        What is asserted is that there is no finite limit. Prove by contradiction. Suppose $lim_{xto 0+}frac 1 {x^{2}}=L$ exists and is finite. Then we can find $delta >0$ such that $|frac 1 {x^{2}}-L|<1$ whenever $0<x<delta$. If $n$ is any sufficiently large integer then $0<frac 1 n<delta$ so, taking $x=frac 1 n$ we get $| n^{2}-L|<1$. This gives $n^{2} <L+1$ for any sufficiently large integer $n$ which is obviously a contradiction.






        share|cite|improve this answer









        $endgroup$



        What is asserted is that there is no finite limit. Prove by contradiction. Suppose $lim_{xto 0+}frac 1 {x^{2}}=L$ exists and is finite. Then we can find $delta >0$ such that $|frac 1 {x^{2}}-L|<1$ whenever $0<x<delta$. If $n$ is any sufficiently large integer then $0<frac 1 n<delta$ so, taking $x=frac 1 n$ we get $| n^{2}-L|<1$. This gives $n^{2} <L+1$ for any sufficiently large integer $n$ which is obviously a contradiction.







        share|cite|improve this answer












        share|cite|improve this answer



        share|cite|improve this answer










        answered Dec 20 '18 at 10:00









        Kavi Rama MurthyKavi Rama Murthy

        60.6k42161




        60.6k42161






























            draft saved

            draft discarded




















































            Thanks for contributing an answer to Mathematics Stack Exchange!


            • Please be sure to answer the question. Provide details and share your research!

            But avoid



            • Asking for help, clarification, or responding to other answers.

            • Making statements based on opinion; back them up with references or personal experience.


            Use MathJax to format equations. MathJax reference.


            To learn more, see our tips on writing great answers.




            draft saved


            draft discarded














            StackExchange.ready(
            function () {
            StackExchange.openid.initPostLogin('.new-post-login', 'https%3a%2f%2fmath.stackexchange.com%2fquestions%2f3047354%2fshow-that-the-limit-does-not-exist-lim-x-to0-frac1x2%23new-answer', 'question_page');
            }
            );

            Post as a guest















            Required, but never shown





















































            Required, but never shown














            Required, but never shown












            Required, but never shown







            Required, but never shown

































            Required, but never shown














            Required, but never shown












            Required, but never shown







            Required, but never shown







            Popular posts from this blog

            Quarter-circle Tiles

            build a pushdown automaton that recognizes the reverse language of a given pushdown automaton?

            Mont Emei